Saturday 12 November 2016

calculus - Will moving differentiation from inside, to outside an integral, change the result?



I'm interested in the potential of such a technique. I got the idea from Moron's answer to this question, which uses the technique of differentiation under the integral.



Now, I'd like to consider this integral:



$$\int_{-\pi}^\pi \cos{(y(1-e^{i\cdot n \cdot t}))}\mathrm dt$$




I'd like to differentiate with respect to y. This will give the integral:



$$\int_{-\pi}^\pi -(1-e^{i\cdot n \cdot t})(\sin{(y(1-e^{i\cdot n \cdot t}))}\mathrm dt$$



...If I'm correct. Anyways, I'm interested in obtaining the results to this second integral, using this technique. So I'm wondering if solving the first integral can help give results for the second integral. I'm thinking of setting $y=1$ in the second integral. This should eliminate $y$ from the result, and give me the integral involving $x$.



The trouble is, I'm not sure I can use the technique of differentiation under the integral. I want to know how I can apply this technique to the integrals above. Any pointers are appreciated.



For instance, for what values of $y$ is this valid?



Answer



Wikipedia doesn't seem to have a precise statement of this theorem. Here's a very general statement.



Theorem (Differentiation under the integral sign): Let $U$ be an open subset of $\mathbb{R}$ and let $E$ be a measure space (which you can freely take to be any open subset of $\mathbb{R}^n$ if you want). Let $f : U \times E \to \mathbb{R}$ have the following properties:




  • $x \mapsto f(t, x)$ is integrable for all $t$,

  • $t \mapsto f(t, x)$ is differentiable for all $x$,

  • for some integrable function $g$, for all $x \in E$, and for all $t \in U$,




$$\left| \frac{\partial f}{\partial t}(t, x) \right| \le g(x).$$



Then the function $x \mapsto \frac{\partial f}{\partial t}(t, x)$ is integrable for all $t$. Moreover, the function $F : U \to \mathbb{R}$ defined by



$$F(t) = \int_E f(t, x) \mu(dx)$$



is differentiable, and



$$F'(t) = \int_E \frac{\partial f}{\partial t}(t, x) \mu(dx).$$




In practice the only condition that isn't easily satisfiable is the third one. It is satisfied in this case, so you're fine (for all $y$).


No comments:

Post a Comment

real analysis - How to find $lim_{hrightarrow 0}frac{sin(ha)}{h}$

How to find $\lim_{h\rightarrow 0}\frac{\sin(ha)}{h}$ without lhopital rule? I know when I use lhopital I easy get $$ \lim_{h\rightarrow 0}...